Lower Bound on Weighted Sum of Auto Correlation

Click For Summary
SUMMARY

The discussion centers on proving the existence of a positive constant α such that the weighted sum of the auto-correlation functions of a sequence v, defined as $$ \sum_{k = - \infty}^{\infty} {2}^{- \left| k \right|} \left \langle {v}^{\left ( 0 \right )}, {v}^{\left ( k \right )} \right \rangle $$, is greater than or equal to α times the squared norm of v. The proof involves utilizing the properties of the auto-correlation function, which is symmetric and hermitian, alongside the convolution theorem. The result is established through the discrete-time Fourier transform (DTFT) of the auto-correlation function and the defined sequence.

PREREQUISITES
  • Understanding of auto-correlation functions and their properties
  • Familiarity with the Cauchy-Schwarz inequality
  • Knowledge of discrete-time Fourier transform (DTFT)
  • Basic principles of convolution and convolution theorem
NEXT STEPS
  • Study the properties of Hermitian functions in signal processing
  • Explore the application of the Cauchy-Schwarz inequality in functional analysis
  • Learn about the implications of the convolution theorem in signal analysis
  • Investigate advanced topics in auto-correlation and its applications in time series analysis
USEFUL FOR

Mathematicians, signal processing engineers, and students studying time series analysis or functional analysis will benefit from this discussion.

Drazick
Messages
10
Reaction score
2

Homework Statement



Given ##v = {\left\{ {v}_{i} \right\}}_{i = 1}^{\infty}## and defining ## {v}_{n}^{\left( k \right)} = {v}_{n - k} ## (Shifting Operator).

Prove that there exist ## \alpha > 0 ## such that

$$ \sum_{k = - \infty}^{\infty} {2}^{- \left| k \right|} \left \langle {v}^{\left ( 0 \right )}, {v}^{\left ( k \right )} \right \rangle \geq \alpha {\left\| v \right\|}^{2} $$

Basically, a Weighted Sum of the Auto Correlation Functions must be greater then its value at Zero.
Namely, the Negative Values can not be summed to match the positive values.

Homework Equations


Probably some variant of Cauchy Schwartz inequality.
Maybe taking advantage of the Auto Correlation function being Symmetric.

The Attempt at a Solution


I understand that behind the scenes the correlations of the shifted versions cant' have negative value which is up to half of the norm of the vector ## v ##.
Yet I couldn't use it to prove the above.

Thank You.
 
Physics news on Phys.org
Defining ## a \left[ k \right] = {2}^{- \left| k \right|} ##.
Moreover, the Auto Correlation function of ## v ## defined as ## {r}_{vv} \left[ k \right] = \left \langle {v}^{\left( 0 \right)}, {v}^{\left( k \right)} \right \rangle = \sum_{n = -\infty}^{\infty} {v}_{n} {v}_{n - k} ##.
Pay attention that [Auto Correlation][1] is [Hermitian Function][2].

Using the definition of [Convolution][3] one could write:

$$ \left( {r}_{vv} \ast a \right) \left[ 0 \right] = \sum_{k = -\infty}^{\infty} {2}^{- \left| k \right| } \left \langle {v}^{\left( 0 \right)}, {v}^{\left( k \right)} \right \rangle $$

Using the [Convolution Theorem][4] one could write that:

$$ \left( {r}_{vv} \ast a \right) \left[ 0 \right] = \int_{- \pi}^{\pi} {R}_{vv} \left( \omega \right) A \left( \omega \right) d \omega $$

Where ## R \left( \omega \right) ## and ## {R}_{vv} \left( \omega \right) ## are the DTFT of ## {r}_{vv} \left[ k \right] ## and ## a \left[ k \right] ## respectively.

One should notice the DTFT of ## a \left[ k \right] ## is defined only one sided. Yet since its symmetrical it can well calculated:

$$
\begin{align*}
A \left( \omega \right) & = DTFT \left\{ a \left[ k \right] \right\} = \sum_{k = -\infty}^{\infty} a \left[ k \right] {e}^{-j \omega k} = \sum_{k = 0}^{\infty} {2}^{-k} {e}^{-j \omega k} + \sum_{k = 0}^{\infty} {2}^{-k} {e}^{j \omega k} - 1 \\
& = \frac{1}{1 - 0.5 {e}^{-j \omega}} + \frac{1}{1 - 0.5 {e}^{j \omega}} - 1 = \frac{1 - {c}^{2}}{1 - 2 c \cos \left( \omega \right) + {c}^{2}} = \alpha > 0 \quad \forall c < 1
\end{align*}
$$

In the above ## c = {2}^{-1} = 0.5 ## yet actually this will hold for any ## c < 1 ##.

So the integral is given by:

$$
\begin{align*}
\int_{- \pi}^{\pi} {R}_{vv} \left( \omega \right) A \left( \omega \right) d \omega & = \int_{- \pi}^{\pi} {R}_{vv} \left( \omega \right) \frac{1 - {c}^{2}}{1 - 2 \alpha \cos \left( \omega \right) + {c}^{2}} d \omega \\
& \geq \alpha \int_{- \pi}^{\pi} {R}_{vv} \left( \omega \right) d \omega = \alpha {\left\| v \right\|}^{2}
\end{align*}
$$

As requested.

By the way the result must be real since ## a \left[ k \right] ## is symmetric and ## {r}_{vv} ## is hermitian function and hence its transform is real.

[1]: https://en.wikipedia.org/wiki/Autocorrelation
[2]: https://en.wikipedia.org/wiki/Hermitian_function
[3]: https://en.wikipedia.org/wiki/Convolution
[4]: https://en.wikipedia.org/wiki/Convolution_theorem
 
Question: A clock's minute hand has length 4 and its hour hand has length 3. What is the distance between the tips at the moment when it is increasing most rapidly?(Putnam Exam Question) Answer: Making assumption that both the hands moves at constant angular velocities, the answer is ## \sqrt{7} .## But don't you think this assumption is somewhat doubtful and wrong?

Similar threads

  • · Replies 3 ·
Replies
3
Views
3K
  • · Replies 2 ·
Replies
2
Views
896
  • · Replies 4 ·
Replies
4
Views
1K
  • · Replies 1 ·
Replies
1
Views
3K
  • · Replies 2 ·
Replies
2
Views
3K
  • · Replies 1 ·
Replies
1
Views
3K
  • · Replies 3 ·
Replies
3
Views
2K
  • · Replies 9 ·
Replies
9
Views
2K
  • · Replies 61 ·
3
Replies
61
Views
10K
  • · Replies 3 ·
Replies
3
Views
2K